Prove for all $a, b, c in mathbb {R^+}$ ${aoversqrt{a+b}} + {boversqrt{b+c}} + {coversqrt{c+a}} gt...












2















Prove for all $a, b, c in mathbb {R^+}$ $${aoversqrt{a+b}} + {boversqrt{b+c}} + {coversqrt{c+a}} gt sqrt{a+b+c}$$ is true




I have been studying about inequalities and how to prove different inequalities, and I stumbled over this question which I failed to answer. I failed to see a connection between $sqrt{a+b+c} text{ and} {aoversqrt{a+b}} + {boversqrt{b+c}} + {coversqrt{c+a}}$. Is there any way I can prove this question and if there is can you please help. Thank you.










share|cite|improve this question



























    2















    Prove for all $a, b, c in mathbb {R^+}$ $${aoversqrt{a+b}} + {boversqrt{b+c}} + {coversqrt{c+a}} gt sqrt{a+b+c}$$ is true




    I have been studying about inequalities and how to prove different inequalities, and I stumbled over this question which I failed to answer. I failed to see a connection between $sqrt{a+b+c} text{ and} {aoversqrt{a+b}} + {boversqrt{b+c}} + {coversqrt{c+a}}$. Is there any way I can prove this question and if there is can you please help. Thank you.










    share|cite|improve this question

























      2












      2








      2


      2






      Prove for all $a, b, c in mathbb {R^+}$ $${aoversqrt{a+b}} + {boversqrt{b+c}} + {coversqrt{c+a}} gt sqrt{a+b+c}$$ is true




      I have been studying about inequalities and how to prove different inequalities, and I stumbled over this question which I failed to answer. I failed to see a connection between $sqrt{a+b+c} text{ and} {aoversqrt{a+b}} + {boversqrt{b+c}} + {coversqrt{c+a}}$. Is there any way I can prove this question and if there is can you please help. Thank you.










      share|cite|improve this question














      Prove for all $a, b, c in mathbb {R^+}$ $${aoversqrt{a+b}} + {boversqrt{b+c}} + {coversqrt{c+a}} gt sqrt{a+b+c}$$ is true




      I have been studying about inequalities and how to prove different inequalities, and I stumbled over this question which I failed to answer. I failed to see a connection between $sqrt{a+b+c} text{ and} {aoversqrt{a+b}} + {boversqrt{b+c}} + {coversqrt{c+a}}$. Is there any way I can prove this question and if there is can you please help. Thank you.







      algebra-precalculus






      share|cite|improve this question













      share|cite|improve this question











      share|cite|improve this question




      share|cite|improve this question










      asked Nov 26 '18 at 6:29









      user587054

      45711




      45711






















          4 Answers
          4






          active

          oldest

          votes


















          1














          You may proceed as follows using convexity of $frac{1}{sqrt{x}}$:





          • $sum_{k=1}^3 lambda_k f(x_k) geq fleft( sum_{k=1}^3 lambda_k x_kright)$ with $sum_{k=1}^3 lambda_k = 1$, $lambda_1,lambda_2,lambda_3 geq 0$
            $$
            begin{eqnarray*} {aoversqrt{a+b}} + {boversqrt{b+c}} + {coversqrt{c+a}}
            & = & (a+b+c)sum_{cyc}frac{a}{(a+b+c)(a+b)} \
            & stackrel{Jensen}{geq} & (a+b+c)frac{1}{sqrt{frac{1}{a+b+c}sum_{cyc}a(a+b)}} \
            & color{blue}{>} & (a+b+c)frac{1}{sqrt{frac{1}{a+b+c}sum_{cyc}a(a+bcolor{blue}{+c})}} \
            & = & (a+b+c)frac{1}{sqrt{frac{1}{a+b+c}(a+b+c)^2}} \
            & = & color{blue}{sqrt{a+b+c}}
            end{eqnarray*}
            $$







          share|cite|improve this answer





























            1














            You need only Jensen applied to convex function $f(x)={1 over sqrt{x}}$:



            $$LHS=aspace f(a+b)+bspace f(b+c)+cspace f(c+a)ge(a+b+c)space f(frac{a(a+b)+b(b+c)+c(c+a)}{a+b+c})$$



            $$LHSge(a+b+c)space f(frac{(a+b+c)^2-ab-bc-ac}{a+b+c})$$



            $$LHSge(a+b+c)sqrt{frac{a+b+c}{(a+b+c)^2-ab-bc-ac}}$$



            $$LHSgt(a+b+c)sqrt{frac{a+b+c}{(a+b+c)^2}}$$



            $$LHSgtsqrt{a+b+c}$$






            share|cite|improve this answer































              0















              Holder's Inequality: Let $a_{ij}, 1 leq i leq m , 1 leq j leq m $ be positive real numbers. Then the following inequality holds:



              $$prod_{i = 1}^{m}left(sum_{j = 1}^{n}a_{ij}right) geq
              left(sum_{j = 1}^{n} sqrt[m]{prod_{i = 1}^{m} a_{ij}}right)^{m}.
              > $$



              For example, consider real numbers $a, b, c, p, q, r, x, y, z$. Then,



              $$(a^{3} + b^{3} + c^{3})(p^{3} + q^{3} + r^{3})(x^{3} + y^{3} +
              > z^{3}) geq (aqx + bqy + crz)^{3}. $$




              We have



              $$sum_{text{cyc}} frac{a}{sqrt{a + b}} > sum_{text{cyc}} frac{a}{sqrt{a + 2b}}. $$



              By Holder's Inequality,



              $$left(sum_{text{cyc}} frac{a}{sqrt{a + 2b}}right)left(sum_{text{cyc}} frac{a}{sqrt{a + 2b}}right)left(sum_{text{cyc}} a(a + 2b)right) geq (a + b+ c)^{3}. $$



              Thus,



              $$left(sum_{text{cyc}}frac{a}{sqrt{a + 2b}}right)^{2} geq a + b + c, $$



              and the result follows.






              share|cite|improve this answer























              • How did you use Holder's Inequality to get that?
                – Vee Hua Zhi
                Nov 26 '18 at 6:58










              • Check my example in the edited post @Tralala
                – Ekesh
                Nov 26 '18 at 7:02










              • There's a typo. $sum_{cyc} a(a+b)$ should be $sum_{cyc} a(a+2b) [= (a+b+c)^2]$.
                – Song
                Nov 26 '18 at 7:20










              • Fixed it. @Song
                – Ekesh
                Nov 26 '18 at 7:24



















              0














              too long; can't fit into a comment. I think @Ekesh 's answer has a little flaw: @Ekesh please see if mine is correct.





              Your inequality is equivalent to



              $$sum_{text{cyc}} frac{a}{sqrt{a + b}}geq a+b+c$$



              By Holder's,



              $$left(sum_{text{cyc}} frac{a}{sqrt{a + b}}right)left(sum_{text{cyc}} frac{a}{sqrt{a + b}}right)left(sum_{text{cyc}} a(a + b)right) geq (a + b+ c)^{3}.$$



              We notice that (by expanding),



              $$left(sum_{text{cyc}} a(a + b)right) leq (a + b+ c)^{2}.$$



              It is valid to divide the above equation with this as $fbox{1}$ It is positive $fbox{2}$ This move removes more value at the RHS, which will not affect the sign.



              Hence we get



              $$left(sum_{text{cyc}}frac{a}{sqrt{a + b}}right)^{2} geq a + b + c,$$



              which is what we want.






              share|cite|improve this answer























              • @Ekesh please read my answer
                – Vee Hua Zhi
                Nov 26 '18 at 7:11










              • yes, your answer is correct.
                – Ekesh
                Nov 26 '18 at 7:23










              • Why downvote? :(
                – Vee Hua Zhi
                Nov 26 '18 at 7:35










              • @Tralala I don't understand that downvote either so I'm fixing that with my upvote :)
                – Oldboy
                Nov 26 '18 at 11:43











              Your Answer





              StackExchange.ifUsing("editor", function () {
              return StackExchange.using("mathjaxEditing", function () {
              StackExchange.MarkdownEditor.creationCallbacks.add(function (editor, postfix) {
              StackExchange.mathjaxEditing.prepareWmdForMathJax(editor, postfix, [["$", "$"], ["\\(","\\)"]]);
              });
              });
              }, "mathjax-editing");

              StackExchange.ready(function() {
              var channelOptions = {
              tags: "".split(" "),
              id: "69"
              };
              initTagRenderer("".split(" "), "".split(" "), channelOptions);

              StackExchange.using("externalEditor", function() {
              // Have to fire editor after snippets, if snippets enabled
              if (StackExchange.settings.snippets.snippetsEnabled) {
              StackExchange.using("snippets", function() {
              createEditor();
              });
              }
              else {
              createEditor();
              }
              });

              function createEditor() {
              StackExchange.prepareEditor({
              heartbeatType: 'answer',
              autoActivateHeartbeat: false,
              convertImagesToLinks: true,
              noModals: true,
              showLowRepImageUploadWarning: true,
              reputationToPostImages: 10,
              bindNavPrevention: true,
              postfix: "",
              imageUploader: {
              brandingHtml: "Powered by u003ca class="icon-imgur-white" href="https://imgur.com/"u003eu003c/au003e",
              contentPolicyHtml: "User contributions licensed under u003ca href="https://creativecommons.org/licenses/by-sa/3.0/"u003ecc by-sa 3.0 with attribution requiredu003c/au003e u003ca href="https://stackoverflow.com/legal/content-policy"u003e(content policy)u003c/au003e",
              allowUrls: true
              },
              noCode: true, onDemand: true,
              discardSelector: ".discard-answer"
              ,immediatelyShowMarkdownHelp:true
              });


              }
              });














              draft saved

              draft discarded


















              StackExchange.ready(
              function () {
              StackExchange.openid.initPostLogin('.new-post-login', 'https%3a%2f%2fmath.stackexchange.com%2fquestions%2f3013932%2fprove-for-all-a-b-c-in-mathbb-r-a-over-sqrtab-b-over-sqrtbc%23new-answer', 'question_page');
              }
              );

              Post as a guest















              Required, but never shown

























              4 Answers
              4






              active

              oldest

              votes








              4 Answers
              4






              active

              oldest

              votes









              active

              oldest

              votes






              active

              oldest

              votes









              1














              You may proceed as follows using convexity of $frac{1}{sqrt{x}}$:





              • $sum_{k=1}^3 lambda_k f(x_k) geq fleft( sum_{k=1}^3 lambda_k x_kright)$ with $sum_{k=1}^3 lambda_k = 1$, $lambda_1,lambda_2,lambda_3 geq 0$
                $$
                begin{eqnarray*} {aoversqrt{a+b}} + {boversqrt{b+c}} + {coversqrt{c+a}}
                & = & (a+b+c)sum_{cyc}frac{a}{(a+b+c)(a+b)} \
                & stackrel{Jensen}{geq} & (a+b+c)frac{1}{sqrt{frac{1}{a+b+c}sum_{cyc}a(a+b)}} \
                & color{blue}{>} & (a+b+c)frac{1}{sqrt{frac{1}{a+b+c}sum_{cyc}a(a+bcolor{blue}{+c})}} \
                & = & (a+b+c)frac{1}{sqrt{frac{1}{a+b+c}(a+b+c)^2}} \
                & = & color{blue}{sqrt{a+b+c}}
                end{eqnarray*}
                $$







              share|cite|improve this answer


























                1














                You may proceed as follows using convexity of $frac{1}{sqrt{x}}$:





                • $sum_{k=1}^3 lambda_k f(x_k) geq fleft( sum_{k=1}^3 lambda_k x_kright)$ with $sum_{k=1}^3 lambda_k = 1$, $lambda_1,lambda_2,lambda_3 geq 0$
                  $$
                  begin{eqnarray*} {aoversqrt{a+b}} + {boversqrt{b+c}} + {coversqrt{c+a}}
                  & = & (a+b+c)sum_{cyc}frac{a}{(a+b+c)(a+b)} \
                  & stackrel{Jensen}{geq} & (a+b+c)frac{1}{sqrt{frac{1}{a+b+c}sum_{cyc}a(a+b)}} \
                  & color{blue}{>} & (a+b+c)frac{1}{sqrt{frac{1}{a+b+c}sum_{cyc}a(a+bcolor{blue}{+c})}} \
                  & = & (a+b+c)frac{1}{sqrt{frac{1}{a+b+c}(a+b+c)^2}} \
                  & = & color{blue}{sqrt{a+b+c}}
                  end{eqnarray*}
                  $$







                share|cite|improve this answer
























                  1












                  1








                  1






                  You may proceed as follows using convexity of $frac{1}{sqrt{x}}$:





                  • $sum_{k=1}^3 lambda_k f(x_k) geq fleft( sum_{k=1}^3 lambda_k x_kright)$ with $sum_{k=1}^3 lambda_k = 1$, $lambda_1,lambda_2,lambda_3 geq 0$
                    $$
                    begin{eqnarray*} {aoversqrt{a+b}} + {boversqrt{b+c}} + {coversqrt{c+a}}
                    & = & (a+b+c)sum_{cyc}frac{a}{(a+b+c)(a+b)} \
                    & stackrel{Jensen}{geq} & (a+b+c)frac{1}{sqrt{frac{1}{a+b+c}sum_{cyc}a(a+b)}} \
                    & color{blue}{>} & (a+b+c)frac{1}{sqrt{frac{1}{a+b+c}sum_{cyc}a(a+bcolor{blue}{+c})}} \
                    & = & (a+b+c)frac{1}{sqrt{frac{1}{a+b+c}(a+b+c)^2}} \
                    & = & color{blue}{sqrt{a+b+c}}
                    end{eqnarray*}
                    $$







                  share|cite|improve this answer












                  You may proceed as follows using convexity of $frac{1}{sqrt{x}}$:





                  • $sum_{k=1}^3 lambda_k f(x_k) geq fleft( sum_{k=1}^3 lambda_k x_kright)$ with $sum_{k=1}^3 lambda_k = 1$, $lambda_1,lambda_2,lambda_3 geq 0$
                    $$
                    begin{eqnarray*} {aoversqrt{a+b}} + {boversqrt{b+c}} + {coversqrt{c+a}}
                    & = & (a+b+c)sum_{cyc}frac{a}{(a+b+c)(a+b)} \
                    & stackrel{Jensen}{geq} & (a+b+c)frac{1}{sqrt{frac{1}{a+b+c}sum_{cyc}a(a+b)}} \
                    & color{blue}{>} & (a+b+c)frac{1}{sqrt{frac{1}{a+b+c}sum_{cyc}a(a+bcolor{blue}{+c})}} \
                    & = & (a+b+c)frac{1}{sqrt{frac{1}{a+b+c}(a+b+c)^2}} \
                    & = & color{blue}{sqrt{a+b+c}}
                    end{eqnarray*}
                    $$








                  share|cite|improve this answer












                  share|cite|improve this answer



                  share|cite|improve this answer










                  answered Nov 26 '18 at 7:10









                  trancelocation

                  9,1151521




                  9,1151521























                      1














                      You need only Jensen applied to convex function $f(x)={1 over sqrt{x}}$:



                      $$LHS=aspace f(a+b)+bspace f(b+c)+cspace f(c+a)ge(a+b+c)space f(frac{a(a+b)+b(b+c)+c(c+a)}{a+b+c})$$



                      $$LHSge(a+b+c)space f(frac{(a+b+c)^2-ab-bc-ac}{a+b+c})$$



                      $$LHSge(a+b+c)sqrt{frac{a+b+c}{(a+b+c)^2-ab-bc-ac}}$$



                      $$LHSgt(a+b+c)sqrt{frac{a+b+c}{(a+b+c)^2}}$$



                      $$LHSgtsqrt{a+b+c}$$






                      share|cite|improve this answer




























                        1














                        You need only Jensen applied to convex function $f(x)={1 over sqrt{x}}$:



                        $$LHS=aspace f(a+b)+bspace f(b+c)+cspace f(c+a)ge(a+b+c)space f(frac{a(a+b)+b(b+c)+c(c+a)}{a+b+c})$$



                        $$LHSge(a+b+c)space f(frac{(a+b+c)^2-ab-bc-ac}{a+b+c})$$



                        $$LHSge(a+b+c)sqrt{frac{a+b+c}{(a+b+c)^2-ab-bc-ac}}$$



                        $$LHSgt(a+b+c)sqrt{frac{a+b+c}{(a+b+c)^2}}$$



                        $$LHSgtsqrt{a+b+c}$$






                        share|cite|improve this answer


























                          1












                          1








                          1






                          You need only Jensen applied to convex function $f(x)={1 over sqrt{x}}$:



                          $$LHS=aspace f(a+b)+bspace f(b+c)+cspace f(c+a)ge(a+b+c)space f(frac{a(a+b)+b(b+c)+c(c+a)}{a+b+c})$$



                          $$LHSge(a+b+c)space f(frac{(a+b+c)^2-ab-bc-ac}{a+b+c})$$



                          $$LHSge(a+b+c)sqrt{frac{a+b+c}{(a+b+c)^2-ab-bc-ac}}$$



                          $$LHSgt(a+b+c)sqrt{frac{a+b+c}{(a+b+c)^2}}$$



                          $$LHSgtsqrt{a+b+c}$$






                          share|cite|improve this answer














                          You need only Jensen applied to convex function $f(x)={1 over sqrt{x}}$:



                          $$LHS=aspace f(a+b)+bspace f(b+c)+cspace f(c+a)ge(a+b+c)space f(frac{a(a+b)+b(b+c)+c(c+a)}{a+b+c})$$



                          $$LHSge(a+b+c)space f(frac{(a+b+c)^2-ab-bc-ac}{a+b+c})$$



                          $$LHSge(a+b+c)sqrt{frac{a+b+c}{(a+b+c)^2-ab-bc-ac}}$$



                          $$LHSgt(a+b+c)sqrt{frac{a+b+c}{(a+b+c)^2}}$$



                          $$LHSgtsqrt{a+b+c}$$







                          share|cite|improve this answer














                          share|cite|improve this answer



                          share|cite|improve this answer








                          edited Nov 26 '18 at 7:24

























                          answered Nov 26 '18 at 7:07









                          Oldboy

                          7,0441832




                          7,0441832























                              0















                              Holder's Inequality: Let $a_{ij}, 1 leq i leq m , 1 leq j leq m $ be positive real numbers. Then the following inequality holds:



                              $$prod_{i = 1}^{m}left(sum_{j = 1}^{n}a_{ij}right) geq
                              left(sum_{j = 1}^{n} sqrt[m]{prod_{i = 1}^{m} a_{ij}}right)^{m}.
                              > $$



                              For example, consider real numbers $a, b, c, p, q, r, x, y, z$. Then,



                              $$(a^{3} + b^{3} + c^{3})(p^{3} + q^{3} + r^{3})(x^{3} + y^{3} +
                              > z^{3}) geq (aqx + bqy + crz)^{3}. $$




                              We have



                              $$sum_{text{cyc}} frac{a}{sqrt{a + b}} > sum_{text{cyc}} frac{a}{sqrt{a + 2b}}. $$



                              By Holder's Inequality,



                              $$left(sum_{text{cyc}} frac{a}{sqrt{a + 2b}}right)left(sum_{text{cyc}} frac{a}{sqrt{a + 2b}}right)left(sum_{text{cyc}} a(a + 2b)right) geq (a + b+ c)^{3}. $$



                              Thus,



                              $$left(sum_{text{cyc}}frac{a}{sqrt{a + 2b}}right)^{2} geq a + b + c, $$



                              and the result follows.






                              share|cite|improve this answer























                              • How did you use Holder's Inequality to get that?
                                – Vee Hua Zhi
                                Nov 26 '18 at 6:58










                              • Check my example in the edited post @Tralala
                                – Ekesh
                                Nov 26 '18 at 7:02










                              • There's a typo. $sum_{cyc} a(a+b)$ should be $sum_{cyc} a(a+2b) [= (a+b+c)^2]$.
                                – Song
                                Nov 26 '18 at 7:20










                              • Fixed it. @Song
                                – Ekesh
                                Nov 26 '18 at 7:24
















                              0















                              Holder's Inequality: Let $a_{ij}, 1 leq i leq m , 1 leq j leq m $ be positive real numbers. Then the following inequality holds:



                              $$prod_{i = 1}^{m}left(sum_{j = 1}^{n}a_{ij}right) geq
                              left(sum_{j = 1}^{n} sqrt[m]{prod_{i = 1}^{m} a_{ij}}right)^{m}.
                              > $$



                              For example, consider real numbers $a, b, c, p, q, r, x, y, z$. Then,



                              $$(a^{3} + b^{3} + c^{3})(p^{3} + q^{3} + r^{3})(x^{3} + y^{3} +
                              > z^{3}) geq (aqx + bqy + crz)^{3}. $$




                              We have



                              $$sum_{text{cyc}} frac{a}{sqrt{a + b}} > sum_{text{cyc}} frac{a}{sqrt{a + 2b}}. $$



                              By Holder's Inequality,



                              $$left(sum_{text{cyc}} frac{a}{sqrt{a + 2b}}right)left(sum_{text{cyc}} frac{a}{sqrt{a + 2b}}right)left(sum_{text{cyc}} a(a + 2b)right) geq (a + b+ c)^{3}. $$



                              Thus,



                              $$left(sum_{text{cyc}}frac{a}{sqrt{a + 2b}}right)^{2} geq a + b + c, $$



                              and the result follows.






                              share|cite|improve this answer























                              • How did you use Holder's Inequality to get that?
                                – Vee Hua Zhi
                                Nov 26 '18 at 6:58










                              • Check my example in the edited post @Tralala
                                – Ekesh
                                Nov 26 '18 at 7:02










                              • There's a typo. $sum_{cyc} a(a+b)$ should be $sum_{cyc} a(a+2b) [= (a+b+c)^2]$.
                                – Song
                                Nov 26 '18 at 7:20










                              • Fixed it. @Song
                                – Ekesh
                                Nov 26 '18 at 7:24














                              0












                              0








                              0







                              Holder's Inequality: Let $a_{ij}, 1 leq i leq m , 1 leq j leq m $ be positive real numbers. Then the following inequality holds:



                              $$prod_{i = 1}^{m}left(sum_{j = 1}^{n}a_{ij}right) geq
                              left(sum_{j = 1}^{n} sqrt[m]{prod_{i = 1}^{m} a_{ij}}right)^{m}.
                              > $$



                              For example, consider real numbers $a, b, c, p, q, r, x, y, z$. Then,



                              $$(a^{3} + b^{3} + c^{3})(p^{3} + q^{3} + r^{3})(x^{3} + y^{3} +
                              > z^{3}) geq (aqx + bqy + crz)^{3}. $$




                              We have



                              $$sum_{text{cyc}} frac{a}{sqrt{a + b}} > sum_{text{cyc}} frac{a}{sqrt{a + 2b}}. $$



                              By Holder's Inequality,



                              $$left(sum_{text{cyc}} frac{a}{sqrt{a + 2b}}right)left(sum_{text{cyc}} frac{a}{sqrt{a + 2b}}right)left(sum_{text{cyc}} a(a + 2b)right) geq (a + b+ c)^{3}. $$



                              Thus,



                              $$left(sum_{text{cyc}}frac{a}{sqrt{a + 2b}}right)^{2} geq a + b + c, $$



                              and the result follows.






                              share|cite|improve this answer















                              Holder's Inequality: Let $a_{ij}, 1 leq i leq m , 1 leq j leq m $ be positive real numbers. Then the following inequality holds:



                              $$prod_{i = 1}^{m}left(sum_{j = 1}^{n}a_{ij}right) geq
                              left(sum_{j = 1}^{n} sqrt[m]{prod_{i = 1}^{m} a_{ij}}right)^{m}.
                              > $$



                              For example, consider real numbers $a, b, c, p, q, r, x, y, z$. Then,



                              $$(a^{3} + b^{3} + c^{3})(p^{3} + q^{3} + r^{3})(x^{3} + y^{3} +
                              > z^{3}) geq (aqx + bqy + crz)^{3}. $$




                              We have



                              $$sum_{text{cyc}} frac{a}{sqrt{a + b}} > sum_{text{cyc}} frac{a}{sqrt{a + 2b}}. $$



                              By Holder's Inequality,



                              $$left(sum_{text{cyc}} frac{a}{sqrt{a + 2b}}right)left(sum_{text{cyc}} frac{a}{sqrt{a + 2b}}right)left(sum_{text{cyc}} a(a + 2b)right) geq (a + b+ c)^{3}. $$



                              Thus,



                              $$left(sum_{text{cyc}}frac{a}{sqrt{a + 2b}}right)^{2} geq a + b + c, $$



                              and the result follows.







                              share|cite|improve this answer














                              share|cite|improve this answer



                              share|cite|improve this answer








                              edited Nov 26 '18 at 7:24

























                              answered Nov 26 '18 at 6:49









                              Ekesh

                              5326




                              5326












                              • How did you use Holder's Inequality to get that?
                                – Vee Hua Zhi
                                Nov 26 '18 at 6:58










                              • Check my example in the edited post @Tralala
                                – Ekesh
                                Nov 26 '18 at 7:02










                              • There's a typo. $sum_{cyc} a(a+b)$ should be $sum_{cyc} a(a+2b) [= (a+b+c)^2]$.
                                – Song
                                Nov 26 '18 at 7:20










                              • Fixed it. @Song
                                – Ekesh
                                Nov 26 '18 at 7:24


















                              • How did you use Holder's Inequality to get that?
                                – Vee Hua Zhi
                                Nov 26 '18 at 6:58










                              • Check my example in the edited post @Tralala
                                – Ekesh
                                Nov 26 '18 at 7:02










                              • There's a typo. $sum_{cyc} a(a+b)$ should be $sum_{cyc} a(a+2b) [= (a+b+c)^2]$.
                                – Song
                                Nov 26 '18 at 7:20










                              • Fixed it. @Song
                                – Ekesh
                                Nov 26 '18 at 7:24
















                              How did you use Holder's Inequality to get that?
                              – Vee Hua Zhi
                              Nov 26 '18 at 6:58




                              How did you use Holder's Inequality to get that?
                              – Vee Hua Zhi
                              Nov 26 '18 at 6:58












                              Check my example in the edited post @Tralala
                              – Ekesh
                              Nov 26 '18 at 7:02




                              Check my example in the edited post @Tralala
                              – Ekesh
                              Nov 26 '18 at 7:02












                              There's a typo. $sum_{cyc} a(a+b)$ should be $sum_{cyc} a(a+2b) [= (a+b+c)^2]$.
                              – Song
                              Nov 26 '18 at 7:20




                              There's a typo. $sum_{cyc} a(a+b)$ should be $sum_{cyc} a(a+2b) [= (a+b+c)^2]$.
                              – Song
                              Nov 26 '18 at 7:20












                              Fixed it. @Song
                              – Ekesh
                              Nov 26 '18 at 7:24




                              Fixed it. @Song
                              – Ekesh
                              Nov 26 '18 at 7:24











                              0














                              too long; can't fit into a comment. I think @Ekesh 's answer has a little flaw: @Ekesh please see if mine is correct.





                              Your inequality is equivalent to



                              $$sum_{text{cyc}} frac{a}{sqrt{a + b}}geq a+b+c$$



                              By Holder's,



                              $$left(sum_{text{cyc}} frac{a}{sqrt{a + b}}right)left(sum_{text{cyc}} frac{a}{sqrt{a + b}}right)left(sum_{text{cyc}} a(a + b)right) geq (a + b+ c)^{3}.$$



                              We notice that (by expanding),



                              $$left(sum_{text{cyc}} a(a + b)right) leq (a + b+ c)^{2}.$$



                              It is valid to divide the above equation with this as $fbox{1}$ It is positive $fbox{2}$ This move removes more value at the RHS, which will not affect the sign.



                              Hence we get



                              $$left(sum_{text{cyc}}frac{a}{sqrt{a + b}}right)^{2} geq a + b + c,$$



                              which is what we want.






                              share|cite|improve this answer























                              • @Ekesh please read my answer
                                – Vee Hua Zhi
                                Nov 26 '18 at 7:11










                              • yes, your answer is correct.
                                – Ekesh
                                Nov 26 '18 at 7:23










                              • Why downvote? :(
                                – Vee Hua Zhi
                                Nov 26 '18 at 7:35










                              • @Tralala I don't understand that downvote either so I'm fixing that with my upvote :)
                                – Oldboy
                                Nov 26 '18 at 11:43
















                              0














                              too long; can't fit into a comment. I think @Ekesh 's answer has a little flaw: @Ekesh please see if mine is correct.





                              Your inequality is equivalent to



                              $$sum_{text{cyc}} frac{a}{sqrt{a + b}}geq a+b+c$$



                              By Holder's,



                              $$left(sum_{text{cyc}} frac{a}{sqrt{a + b}}right)left(sum_{text{cyc}} frac{a}{sqrt{a + b}}right)left(sum_{text{cyc}} a(a + b)right) geq (a + b+ c)^{3}.$$



                              We notice that (by expanding),



                              $$left(sum_{text{cyc}} a(a + b)right) leq (a + b+ c)^{2}.$$



                              It is valid to divide the above equation with this as $fbox{1}$ It is positive $fbox{2}$ This move removes more value at the RHS, which will not affect the sign.



                              Hence we get



                              $$left(sum_{text{cyc}}frac{a}{sqrt{a + b}}right)^{2} geq a + b + c,$$



                              which is what we want.






                              share|cite|improve this answer























                              • @Ekesh please read my answer
                                – Vee Hua Zhi
                                Nov 26 '18 at 7:11










                              • yes, your answer is correct.
                                – Ekesh
                                Nov 26 '18 at 7:23










                              • Why downvote? :(
                                – Vee Hua Zhi
                                Nov 26 '18 at 7:35










                              • @Tralala I don't understand that downvote either so I'm fixing that with my upvote :)
                                – Oldboy
                                Nov 26 '18 at 11:43














                              0












                              0








                              0






                              too long; can't fit into a comment. I think @Ekesh 's answer has a little flaw: @Ekesh please see if mine is correct.





                              Your inequality is equivalent to



                              $$sum_{text{cyc}} frac{a}{sqrt{a + b}}geq a+b+c$$



                              By Holder's,



                              $$left(sum_{text{cyc}} frac{a}{sqrt{a + b}}right)left(sum_{text{cyc}} frac{a}{sqrt{a + b}}right)left(sum_{text{cyc}} a(a + b)right) geq (a + b+ c)^{3}.$$



                              We notice that (by expanding),



                              $$left(sum_{text{cyc}} a(a + b)right) leq (a + b+ c)^{2}.$$



                              It is valid to divide the above equation with this as $fbox{1}$ It is positive $fbox{2}$ This move removes more value at the RHS, which will not affect the sign.



                              Hence we get



                              $$left(sum_{text{cyc}}frac{a}{sqrt{a + b}}right)^{2} geq a + b + c,$$



                              which is what we want.






                              share|cite|improve this answer














                              too long; can't fit into a comment. I think @Ekesh 's answer has a little flaw: @Ekesh please see if mine is correct.





                              Your inequality is equivalent to



                              $$sum_{text{cyc}} frac{a}{sqrt{a + b}}geq a+b+c$$



                              By Holder's,



                              $$left(sum_{text{cyc}} frac{a}{sqrt{a + b}}right)left(sum_{text{cyc}} frac{a}{sqrt{a + b}}right)left(sum_{text{cyc}} a(a + b)right) geq (a + b+ c)^{3}.$$



                              We notice that (by expanding),



                              $$left(sum_{text{cyc}} a(a + b)right) leq (a + b+ c)^{2}.$$



                              It is valid to divide the above equation with this as $fbox{1}$ It is positive $fbox{2}$ This move removes more value at the RHS, which will not affect the sign.



                              Hence we get



                              $$left(sum_{text{cyc}}frac{a}{sqrt{a + b}}right)^{2} geq a + b + c,$$



                              which is what we want.







                              share|cite|improve this answer














                              share|cite|improve this answer



                              share|cite|improve this answer








                              edited Nov 26 '18 at 8:37

























                              answered Nov 26 '18 at 7:11









                              Vee Hua Zhi

                              759224




                              759224












                              • @Ekesh please read my answer
                                – Vee Hua Zhi
                                Nov 26 '18 at 7:11










                              • yes, your answer is correct.
                                – Ekesh
                                Nov 26 '18 at 7:23










                              • Why downvote? :(
                                – Vee Hua Zhi
                                Nov 26 '18 at 7:35










                              • @Tralala I don't understand that downvote either so I'm fixing that with my upvote :)
                                – Oldboy
                                Nov 26 '18 at 11:43


















                              • @Ekesh please read my answer
                                – Vee Hua Zhi
                                Nov 26 '18 at 7:11










                              • yes, your answer is correct.
                                – Ekesh
                                Nov 26 '18 at 7:23










                              • Why downvote? :(
                                – Vee Hua Zhi
                                Nov 26 '18 at 7:35










                              • @Tralala I don't understand that downvote either so I'm fixing that with my upvote :)
                                – Oldboy
                                Nov 26 '18 at 11:43
















                              @Ekesh please read my answer
                              – Vee Hua Zhi
                              Nov 26 '18 at 7:11




                              @Ekesh please read my answer
                              – Vee Hua Zhi
                              Nov 26 '18 at 7:11












                              yes, your answer is correct.
                              – Ekesh
                              Nov 26 '18 at 7:23




                              yes, your answer is correct.
                              – Ekesh
                              Nov 26 '18 at 7:23












                              Why downvote? :(
                              – Vee Hua Zhi
                              Nov 26 '18 at 7:35




                              Why downvote? :(
                              – Vee Hua Zhi
                              Nov 26 '18 at 7:35












                              @Tralala I don't understand that downvote either so I'm fixing that with my upvote :)
                              – Oldboy
                              Nov 26 '18 at 11:43




                              @Tralala I don't understand that downvote either so I'm fixing that with my upvote :)
                              – Oldboy
                              Nov 26 '18 at 11:43


















                              draft saved

                              draft discarded




















































                              Thanks for contributing an answer to Mathematics Stack Exchange!


                              • Please be sure to answer the question. Provide details and share your research!

                              But avoid



                              • Asking for help, clarification, or responding to other answers.

                              • Making statements based on opinion; back them up with references or personal experience.


                              Use MathJax to format equations. MathJax reference.


                              To learn more, see our tips on writing great answers.





                              Some of your past answers have not been well-received, and you're in danger of being blocked from answering.


                              Please pay close attention to the following guidance:


                              • Please be sure to answer the question. Provide details and share your research!

                              But avoid



                              • Asking for help, clarification, or responding to other answers.

                              • Making statements based on opinion; back them up with references or personal experience.


                              To learn more, see our tips on writing great answers.




                              draft saved


                              draft discarded














                              StackExchange.ready(
                              function () {
                              StackExchange.openid.initPostLogin('.new-post-login', 'https%3a%2f%2fmath.stackexchange.com%2fquestions%2f3013932%2fprove-for-all-a-b-c-in-mathbb-r-a-over-sqrtab-b-over-sqrtbc%23new-answer', 'question_page');
                              }
                              );

                              Post as a guest















                              Required, but never shown





















































                              Required, but never shown














                              Required, but never shown












                              Required, but never shown







                              Required, but never shown

































                              Required, but never shown














                              Required, but never shown












                              Required, but never shown







                              Required, but never shown







                              Popular posts from this blog

                              Plaza Victoria

                              In PowerPoint, is there a keyboard shortcut for bulleted / numbered list?

                              How to put 3 figures in Latex with 2 figures side by side and 1 below these side by side images but in...